Describe the Natural density of $p$ which divides natural numbers of the form $n^2+1$?Prime Divisors of $x^2 + 1$Show that the odd prime divisors of $n^2+1$ are of form $4k+1$“If $m$ divides two Fermat numbers, $m$ divides $2$.” Why?Describe the set of all good numbersWhat is the smallest natural number divisible by the first $n$ natural numbers?Prove that every odd natural number divides some number of the form $2^n - 1$Mental Primality TestingNumbers divisible by all of their digits: Why don't 4's show up in 6- or 7- digit numbers?Any composite natural number divides the product of two smaller natural numbersFractals using just modulo operationDescribe all natural numbers $n$ for which $3^n-2^n$ is divisible by $5$.Set of $n$ natural numbers $a_i$ such that: if $a_jlt a_k$, then $(a_k-a_j)mid a_j$

Is there really no realistic way for a skeleton monster to move around without magic?

GPS Rollover on Android Smartphones

Is Social Media Science Fiction?

How can I fix this gap between bookcases I made?

Do airline pilots ever risk not hearing communication directed to them specifically, from traffic controllers?

Why don't electron-positron collisions release infinite energy?

DOS, create pipe for stdin/stdout of command.com(or 4dos.com) in C or Batch?

What is the command to reset a PC without deleting any files

Why CLRS example on residual networks does not follows its formula?

How can I hide my bitcoin transactions to protect anonymity from others?

What is the logic behind how bash tests for true/false?

Copenhagen passport control - US citizen

A newer friend of my brother's gave him a load of baseball cards that are supposedly extremely valuable. Is this a scam?

How do we improve the relationship with a client software team that performs poorly and is becoming less collaborative?

How is this relation reflexive?

I probably found a bug with the sudo apt install function

How is it possible to have an ability score that is less than 3?

Can a German sentence have two subjects?

New order #4: World

Are there any consumables that function as addictive (psychedelic) drugs?

Set-theoretical foundations of Mathematics with only bounded quantifiers

What typically incentivizes a professor to change jobs to a lower ranking university?

Why is the design of haulage companies so “special”?

Is it possible to do 50 km distance without any previous training?



Describe the Natural density of $p$ which divides natural numbers of the form $n^2+1$?


Prime Divisors of $x^2 + 1$Show that the odd prime divisors of $n^2+1$ are of form $4k+1$“If $m$ divides two Fermat numbers, $m$ divides $2$.” Why?Describe the set of all good numbersWhat is the smallest natural number divisible by the first $n$ natural numbers?Prove that every odd natural number divides some number of the form $2^n - 1$Mental Primality TestingNumbers divisible by all of their digits: Why don't 4's show up in 6- or 7- digit numbers?Any composite natural number divides the product of two smaller natural numbersFractals using just modulo operationDescribe all natural numbers $n$ for which $3^n-2^n$ is divisible by $5$.Set of $n$ natural numbers $a_i$ such that: if $a_jlt a_k$, then $(a_k-a_j)mid a_j$













1












$begingroup$


We want to find the numbers that divide natural numbers in the form of $n^2+1$ and solve for their natural density.



Using Wolfram Mathematica, I found divisors from $n=0$ to $1000000$ and eliminated repeated divisors. Here is the list



$$left1,2,5,10,13,17,25,26,29,34,37,41,50,53,58,61,65,73,74,82,85,89,97,101,106,109,113,122,125,130,137,145,146,149,157,169,170,173,178,181,185,193,194,197,202,205,218,221,226,229,233,241,250,257,265,269,274,277,281..... right$$



After looking at the list extensively my guess is the density is zero.



Is there a mathematical way of finding this without computer programming?










share|cite|improve this question











$endgroup$











  • $begingroup$
    Comments are not for extended discussion; this conversation has been moved to chat.
    $endgroup$
    – Aloizio Macedo
    Mar 28 at 12:44










  • $begingroup$
    @AloizioMacedo Why is my question still unclear?
    $endgroup$
    – Arbuja
    Mar 28 at 21:15






  • 1




    $begingroup$
    The divisors of $n^2+1$ are precisely the numbers not divisible by four and with no prime factor one less than a multiple of four. These are tabulated at oeis.org/A008784 and there are links there which may lead to an answer to your question.
    $endgroup$
    – Gerry Myerson
    Mar 29 at 1:52










  • $begingroup$
    Related: math.stackexchange.com/questions/1472923/prime-divisors-of-x2-1 and math.stackexchange.com/questions/926561/…
    $endgroup$
    – Gerry Myerson
    Mar 29 at 1:57






  • 3




    $begingroup$
    I would suggest eliminating your initial general question, which is hopelessly broad. The specific question about numbers of the form $n^2+1$ is a perfectly good question though.
    $endgroup$
    – Eric Wofsey
    Mar 29 at 3:52















1












$begingroup$


We want to find the numbers that divide natural numbers in the form of $n^2+1$ and solve for their natural density.



Using Wolfram Mathematica, I found divisors from $n=0$ to $1000000$ and eliminated repeated divisors. Here is the list



$$left1,2,5,10,13,17,25,26,29,34,37,41,50,53,58,61,65,73,74,82,85,89,97,101,106,109,113,122,125,130,137,145,146,149,157,169,170,173,178,181,185,193,194,197,202,205,218,221,226,229,233,241,250,257,265,269,274,277,281..... right$$



After looking at the list extensively my guess is the density is zero.



Is there a mathematical way of finding this without computer programming?










share|cite|improve this question











$endgroup$











  • $begingroup$
    Comments are not for extended discussion; this conversation has been moved to chat.
    $endgroup$
    – Aloizio Macedo
    Mar 28 at 12:44










  • $begingroup$
    @AloizioMacedo Why is my question still unclear?
    $endgroup$
    – Arbuja
    Mar 28 at 21:15






  • 1




    $begingroup$
    The divisors of $n^2+1$ are precisely the numbers not divisible by four and with no prime factor one less than a multiple of four. These are tabulated at oeis.org/A008784 and there are links there which may lead to an answer to your question.
    $endgroup$
    – Gerry Myerson
    Mar 29 at 1:52










  • $begingroup$
    Related: math.stackexchange.com/questions/1472923/prime-divisors-of-x2-1 and math.stackexchange.com/questions/926561/…
    $endgroup$
    – Gerry Myerson
    Mar 29 at 1:57






  • 3




    $begingroup$
    I would suggest eliminating your initial general question, which is hopelessly broad. The specific question about numbers of the form $n^2+1$ is a perfectly good question though.
    $endgroup$
    – Eric Wofsey
    Mar 29 at 3:52













1












1








1


0



$begingroup$


We want to find the numbers that divide natural numbers in the form of $n^2+1$ and solve for their natural density.



Using Wolfram Mathematica, I found divisors from $n=0$ to $1000000$ and eliminated repeated divisors. Here is the list



$$left1,2,5,10,13,17,25,26,29,34,37,41,50,53,58,61,65,73,74,82,85,89,97,101,106,109,113,122,125,130,137,145,146,149,157,169,170,173,178,181,185,193,194,197,202,205,218,221,226,229,233,241,250,257,265,269,274,277,281..... right$$



After looking at the list extensively my guess is the density is zero.



Is there a mathematical way of finding this without computer programming?










share|cite|improve this question











$endgroup$




We want to find the numbers that divide natural numbers in the form of $n^2+1$ and solve for their natural density.



Using Wolfram Mathematica, I found divisors from $n=0$ to $1000000$ and eliminated repeated divisors. Here is the list



$$left1,2,5,10,13,17,25,26,29,34,37,41,50,53,58,61,65,73,74,82,85,89,97,101,106,109,113,122,125,130,137,145,146,149,157,169,170,173,178,181,185,193,194,197,202,205,218,221,226,229,233,241,250,257,265,269,274,277,281..... right$$



After looking at the list extensively my guess is the density is zero.



Is there a mathematical way of finding this without computer programming?







divisibility






share|cite|improve this question















share|cite|improve this question













share|cite|improve this question




share|cite|improve this question








edited Mar 29 at 17:48







Arbuja

















asked Mar 26 at 12:16









ArbujaArbuja

883831




883831











  • $begingroup$
    Comments are not for extended discussion; this conversation has been moved to chat.
    $endgroup$
    – Aloizio Macedo
    Mar 28 at 12:44










  • $begingroup$
    @AloizioMacedo Why is my question still unclear?
    $endgroup$
    – Arbuja
    Mar 28 at 21:15






  • 1




    $begingroup$
    The divisors of $n^2+1$ are precisely the numbers not divisible by four and with no prime factor one less than a multiple of four. These are tabulated at oeis.org/A008784 and there are links there which may lead to an answer to your question.
    $endgroup$
    – Gerry Myerson
    Mar 29 at 1:52










  • $begingroup$
    Related: math.stackexchange.com/questions/1472923/prime-divisors-of-x2-1 and math.stackexchange.com/questions/926561/…
    $endgroup$
    – Gerry Myerson
    Mar 29 at 1:57






  • 3




    $begingroup$
    I would suggest eliminating your initial general question, which is hopelessly broad. The specific question about numbers of the form $n^2+1$ is a perfectly good question though.
    $endgroup$
    – Eric Wofsey
    Mar 29 at 3:52
















  • $begingroup$
    Comments are not for extended discussion; this conversation has been moved to chat.
    $endgroup$
    – Aloizio Macedo
    Mar 28 at 12:44










  • $begingroup$
    @AloizioMacedo Why is my question still unclear?
    $endgroup$
    – Arbuja
    Mar 28 at 21:15






  • 1




    $begingroup$
    The divisors of $n^2+1$ are precisely the numbers not divisible by four and with no prime factor one less than a multiple of four. These are tabulated at oeis.org/A008784 and there are links there which may lead to an answer to your question.
    $endgroup$
    – Gerry Myerson
    Mar 29 at 1:52










  • $begingroup$
    Related: math.stackexchange.com/questions/1472923/prime-divisors-of-x2-1 and math.stackexchange.com/questions/926561/…
    $endgroup$
    – Gerry Myerson
    Mar 29 at 1:57






  • 3




    $begingroup$
    I would suggest eliminating your initial general question, which is hopelessly broad. The specific question about numbers of the form $n^2+1$ is a perfectly good question though.
    $endgroup$
    – Eric Wofsey
    Mar 29 at 3:52















$begingroup$
Comments are not for extended discussion; this conversation has been moved to chat.
$endgroup$
– Aloizio Macedo
Mar 28 at 12:44




$begingroup$
Comments are not for extended discussion; this conversation has been moved to chat.
$endgroup$
– Aloizio Macedo
Mar 28 at 12:44












$begingroup$
@AloizioMacedo Why is my question still unclear?
$endgroup$
– Arbuja
Mar 28 at 21:15




$begingroup$
@AloizioMacedo Why is my question still unclear?
$endgroup$
– Arbuja
Mar 28 at 21:15




1




1




$begingroup$
The divisors of $n^2+1$ are precisely the numbers not divisible by four and with no prime factor one less than a multiple of four. These are tabulated at oeis.org/A008784 and there are links there which may lead to an answer to your question.
$endgroup$
– Gerry Myerson
Mar 29 at 1:52




$begingroup$
The divisors of $n^2+1$ are precisely the numbers not divisible by four and with no prime factor one less than a multiple of four. These are tabulated at oeis.org/A008784 and there are links there which may lead to an answer to your question.
$endgroup$
– Gerry Myerson
Mar 29 at 1:52












$begingroup$
Related: math.stackexchange.com/questions/1472923/prime-divisors-of-x2-1 and math.stackexchange.com/questions/926561/…
$endgroup$
– Gerry Myerson
Mar 29 at 1:57




$begingroup$
Related: math.stackexchange.com/questions/1472923/prime-divisors-of-x2-1 and math.stackexchange.com/questions/926561/…
$endgroup$
– Gerry Myerson
Mar 29 at 1:57




3




3




$begingroup$
I would suggest eliminating your initial general question, which is hopelessly broad. The specific question about numbers of the form $n^2+1$ is a perfectly good question though.
$endgroup$
– Eric Wofsey
Mar 29 at 3:52




$begingroup$
I would suggest eliminating your initial general question, which is hopelessly broad. The specific question about numbers of the form $n^2+1$ is a perfectly good question though.
$endgroup$
– Eric Wofsey
Mar 29 at 3:52










2 Answers
2






active

oldest

votes


















3












$begingroup$

The set in question, call it $A$, is a proper subset of the set, call it $B$, of numbers which can be written as a sum of two squares (the difference between the two sets is that $B$ is the set of all numbers of the form $2^rPQ^2$ where $r$ is a nonnegative integer, the prime factors of $P$ are all $1bmod4$, and the prime factors of $Q$ are all $3bmod4$; $A$ is the same, but with $rle1$ and $Q=1$).



$B$ has density zero (and so, a fortiori, $A$ has density zero). This is discussed at https://mathoverflow.net/questions/205862/sums-of-two-squares-positive-lower-density and at the links to be found there. It goes back to Landau.






share|cite|improve this answer











$endgroup$




















    3












    $begingroup$

    As I understand it, you claim that every divisor of a number of the form $n^2+1$ belongs to one of some set of residue classes (mod 72).



    This appears to be true. For instance if any multiple of 3 were to divide $n^2+1$ then we would have $n^2+1 equiv 0 pmod3$ and in turn $n^2 equiv -1 pmod3$. But -1 is not a quadratic residue (mod 3), so we can rule out classes like 72m, 72m+3, etc. as potential divisors of $n^2+1$.



    I didn't verify your list, but clearly some such list exists.



    If you wanted to go from here to calculating the asymptotic density, you'd need to prove something stronger, say that every (perhaps sufficiently large) number belonging to such a class divides some number of the form $n^2+1$.



    UPDATE



    In fact let's consider that question. Say you have some number $72m + r$ and claim it divides a number of the form $n^2 + 1$. That is, for some k we have $(72m + r)k = n^2 + 1$. Then $-1 + (72m+r)k$ is a perfect square for some $k>0$. But this is the same as $-1$ being a quadratic residue $pmod72m+r$.



    My elementary number theory is rusty, but I think that's equivalent to every odd prime divisor of $72m+r$ being of the form $1 pmod4$. So you could try and see whether this applies to all numbers in such a class.






    share|cite|improve this answer











    $endgroup$








    • 1




      $begingroup$
      It is a valiant effort to make sense of the Question, but the considerations you give do not help in determining "density". Showing that certain residues mod $72$ are possible as divisors of numbers of the form $n^2+1$ does not by itself guarantee all such residues as asymptotically equally likely. I applaud your effort to reason on this, but I'm voting to close the Question as the OP has twice asked pretty much the same ill-defined problem.
      $endgroup$
      – hardmath
      Mar 26 at 18:35










    • $begingroup$
      @hardmath lulu in the discussion, understands my question. What is still unclear?
      $endgroup$
      – Arbuja
      Mar 28 at 22:44






    • 1




      $begingroup$
      @Arbuja: With more recent edits to your Question, I'm now able to upvote both it and Daniel's Answer.
      $endgroup$
      – hardmath
      Mar 30 at 0:31











    Your Answer





    StackExchange.ifUsing("editor", function ()
    return StackExchange.using("mathjaxEditing", function ()
    StackExchange.MarkdownEditor.creationCallbacks.add(function (editor, postfix)
    StackExchange.mathjaxEditing.prepareWmdForMathJax(editor, postfix, [["$", "$"], ["\\(","\\)"]]);
    );
    );
    , "mathjax-editing");

    StackExchange.ready(function()
    var channelOptions =
    tags: "".split(" "),
    id: "69"
    ;
    initTagRenderer("".split(" "), "".split(" "), channelOptions);

    StackExchange.using("externalEditor", function()
    // Have to fire editor after snippets, if snippets enabled
    if (StackExchange.settings.snippets.snippetsEnabled)
    StackExchange.using("snippets", function()
    createEditor();
    );

    else
    createEditor();

    );

    function createEditor()
    StackExchange.prepareEditor(
    heartbeatType: 'answer',
    autoActivateHeartbeat: false,
    convertImagesToLinks: true,
    noModals: true,
    showLowRepImageUploadWarning: true,
    reputationToPostImages: 10,
    bindNavPrevention: true,
    postfix: "",
    imageUploader:
    brandingHtml: "Powered by u003ca class="icon-imgur-white" href="https://imgur.com/"u003eu003c/au003e",
    contentPolicyHtml: "User contributions licensed under u003ca href="https://creativecommons.org/licenses/by-sa/3.0/"u003ecc by-sa 3.0 with attribution requiredu003c/au003e u003ca href="https://stackoverflow.com/legal/content-policy"u003e(content policy)u003c/au003e",
    allowUrls: true
    ,
    noCode: true, onDemand: true,
    discardSelector: ".discard-answer"
    ,immediatelyShowMarkdownHelp:true
    );



    );













    draft saved

    draft discarded


















    StackExchange.ready(
    function ()
    StackExchange.openid.initPostLogin('.new-post-login', 'https%3a%2f%2fmath.stackexchange.com%2fquestions%2f3163100%2fdescribe-the-natural-density-of-p-which-divides-natural-numbers-of-the-form-n%23new-answer', 'question_page');

    );

    Post as a guest















    Required, but never shown

























    2 Answers
    2






    active

    oldest

    votes








    2 Answers
    2






    active

    oldest

    votes









    active

    oldest

    votes






    active

    oldest

    votes









    3












    $begingroup$

    The set in question, call it $A$, is a proper subset of the set, call it $B$, of numbers which can be written as a sum of two squares (the difference between the two sets is that $B$ is the set of all numbers of the form $2^rPQ^2$ where $r$ is a nonnegative integer, the prime factors of $P$ are all $1bmod4$, and the prime factors of $Q$ are all $3bmod4$; $A$ is the same, but with $rle1$ and $Q=1$).



    $B$ has density zero (and so, a fortiori, $A$ has density zero). This is discussed at https://mathoverflow.net/questions/205862/sums-of-two-squares-positive-lower-density and at the links to be found there. It goes back to Landau.






    share|cite|improve this answer











    $endgroup$

















      3












      $begingroup$

      The set in question, call it $A$, is a proper subset of the set, call it $B$, of numbers which can be written as a sum of two squares (the difference between the two sets is that $B$ is the set of all numbers of the form $2^rPQ^2$ where $r$ is a nonnegative integer, the prime factors of $P$ are all $1bmod4$, and the prime factors of $Q$ are all $3bmod4$; $A$ is the same, but with $rle1$ and $Q=1$).



      $B$ has density zero (and so, a fortiori, $A$ has density zero). This is discussed at https://mathoverflow.net/questions/205862/sums-of-two-squares-positive-lower-density and at the links to be found there. It goes back to Landau.






      share|cite|improve this answer











      $endgroup$















        3












        3








        3





        $begingroup$

        The set in question, call it $A$, is a proper subset of the set, call it $B$, of numbers which can be written as a sum of two squares (the difference between the two sets is that $B$ is the set of all numbers of the form $2^rPQ^2$ where $r$ is a nonnegative integer, the prime factors of $P$ are all $1bmod4$, and the prime factors of $Q$ are all $3bmod4$; $A$ is the same, but with $rle1$ and $Q=1$).



        $B$ has density zero (and so, a fortiori, $A$ has density zero). This is discussed at https://mathoverflow.net/questions/205862/sums-of-two-squares-positive-lower-density and at the links to be found there. It goes back to Landau.






        share|cite|improve this answer











        $endgroup$



        The set in question, call it $A$, is a proper subset of the set, call it $B$, of numbers which can be written as a sum of two squares (the difference between the two sets is that $B$ is the set of all numbers of the form $2^rPQ^2$ where $r$ is a nonnegative integer, the prime factors of $P$ are all $1bmod4$, and the prime factors of $Q$ are all $3bmod4$; $A$ is the same, but with $rle1$ and $Q=1$).



        $B$ has density zero (and so, a fortiori, $A$ has density zero). This is discussed at https://mathoverflow.net/questions/205862/sums-of-two-squares-positive-lower-density and at the links to be found there. It goes back to Landau.







        share|cite|improve this answer














        share|cite|improve this answer



        share|cite|improve this answer








        edited Mar 30 at 3:25

























        answered Mar 29 at 23:14









        Gerry MyersonGerry Myerson

        148k8152306




        148k8152306





















            3












            $begingroup$

            As I understand it, you claim that every divisor of a number of the form $n^2+1$ belongs to one of some set of residue classes (mod 72).



            This appears to be true. For instance if any multiple of 3 were to divide $n^2+1$ then we would have $n^2+1 equiv 0 pmod3$ and in turn $n^2 equiv -1 pmod3$. But -1 is not a quadratic residue (mod 3), so we can rule out classes like 72m, 72m+3, etc. as potential divisors of $n^2+1$.



            I didn't verify your list, but clearly some such list exists.



            If you wanted to go from here to calculating the asymptotic density, you'd need to prove something stronger, say that every (perhaps sufficiently large) number belonging to such a class divides some number of the form $n^2+1$.



            UPDATE



            In fact let's consider that question. Say you have some number $72m + r$ and claim it divides a number of the form $n^2 + 1$. That is, for some k we have $(72m + r)k = n^2 + 1$. Then $-1 + (72m+r)k$ is a perfect square for some $k>0$. But this is the same as $-1$ being a quadratic residue $pmod72m+r$.



            My elementary number theory is rusty, but I think that's equivalent to every odd prime divisor of $72m+r$ being of the form $1 pmod4$. So you could try and see whether this applies to all numbers in such a class.






            share|cite|improve this answer











            $endgroup$








            • 1




              $begingroup$
              It is a valiant effort to make sense of the Question, but the considerations you give do not help in determining "density". Showing that certain residues mod $72$ are possible as divisors of numbers of the form $n^2+1$ does not by itself guarantee all such residues as asymptotically equally likely. I applaud your effort to reason on this, but I'm voting to close the Question as the OP has twice asked pretty much the same ill-defined problem.
              $endgroup$
              – hardmath
              Mar 26 at 18:35










            • $begingroup$
              @hardmath lulu in the discussion, understands my question. What is still unclear?
              $endgroup$
              – Arbuja
              Mar 28 at 22:44






            • 1




              $begingroup$
              @Arbuja: With more recent edits to your Question, I'm now able to upvote both it and Daniel's Answer.
              $endgroup$
              – hardmath
              Mar 30 at 0:31















            3












            $begingroup$

            As I understand it, you claim that every divisor of a number of the form $n^2+1$ belongs to one of some set of residue classes (mod 72).



            This appears to be true. For instance if any multiple of 3 were to divide $n^2+1$ then we would have $n^2+1 equiv 0 pmod3$ and in turn $n^2 equiv -1 pmod3$. But -1 is not a quadratic residue (mod 3), so we can rule out classes like 72m, 72m+3, etc. as potential divisors of $n^2+1$.



            I didn't verify your list, but clearly some such list exists.



            If you wanted to go from here to calculating the asymptotic density, you'd need to prove something stronger, say that every (perhaps sufficiently large) number belonging to such a class divides some number of the form $n^2+1$.



            UPDATE



            In fact let's consider that question. Say you have some number $72m + r$ and claim it divides a number of the form $n^2 + 1$. That is, for some k we have $(72m + r)k = n^2 + 1$. Then $-1 + (72m+r)k$ is a perfect square for some $k>0$. But this is the same as $-1$ being a quadratic residue $pmod72m+r$.



            My elementary number theory is rusty, but I think that's equivalent to every odd prime divisor of $72m+r$ being of the form $1 pmod4$. So you could try and see whether this applies to all numbers in such a class.






            share|cite|improve this answer











            $endgroup$








            • 1




              $begingroup$
              It is a valiant effort to make sense of the Question, but the considerations you give do not help in determining "density". Showing that certain residues mod $72$ are possible as divisors of numbers of the form $n^2+1$ does not by itself guarantee all such residues as asymptotically equally likely. I applaud your effort to reason on this, but I'm voting to close the Question as the OP has twice asked pretty much the same ill-defined problem.
              $endgroup$
              – hardmath
              Mar 26 at 18:35










            • $begingroup$
              @hardmath lulu in the discussion, understands my question. What is still unclear?
              $endgroup$
              – Arbuja
              Mar 28 at 22:44






            • 1




              $begingroup$
              @Arbuja: With more recent edits to your Question, I'm now able to upvote both it and Daniel's Answer.
              $endgroup$
              – hardmath
              Mar 30 at 0:31













            3












            3








            3





            $begingroup$

            As I understand it, you claim that every divisor of a number of the form $n^2+1$ belongs to one of some set of residue classes (mod 72).



            This appears to be true. For instance if any multiple of 3 were to divide $n^2+1$ then we would have $n^2+1 equiv 0 pmod3$ and in turn $n^2 equiv -1 pmod3$. But -1 is not a quadratic residue (mod 3), so we can rule out classes like 72m, 72m+3, etc. as potential divisors of $n^2+1$.



            I didn't verify your list, but clearly some such list exists.



            If you wanted to go from here to calculating the asymptotic density, you'd need to prove something stronger, say that every (perhaps sufficiently large) number belonging to such a class divides some number of the form $n^2+1$.



            UPDATE



            In fact let's consider that question. Say you have some number $72m + r$ and claim it divides a number of the form $n^2 + 1$. That is, for some k we have $(72m + r)k = n^2 + 1$. Then $-1 + (72m+r)k$ is a perfect square for some $k>0$. But this is the same as $-1$ being a quadratic residue $pmod72m+r$.



            My elementary number theory is rusty, but I think that's equivalent to every odd prime divisor of $72m+r$ being of the form $1 pmod4$. So you could try and see whether this applies to all numbers in such a class.






            share|cite|improve this answer











            $endgroup$



            As I understand it, you claim that every divisor of a number of the form $n^2+1$ belongs to one of some set of residue classes (mod 72).



            This appears to be true. For instance if any multiple of 3 were to divide $n^2+1$ then we would have $n^2+1 equiv 0 pmod3$ and in turn $n^2 equiv -1 pmod3$. But -1 is not a quadratic residue (mod 3), so we can rule out classes like 72m, 72m+3, etc. as potential divisors of $n^2+1$.



            I didn't verify your list, but clearly some such list exists.



            If you wanted to go from here to calculating the asymptotic density, you'd need to prove something stronger, say that every (perhaps sufficiently large) number belonging to such a class divides some number of the form $n^2+1$.



            UPDATE



            In fact let's consider that question. Say you have some number $72m + r$ and claim it divides a number of the form $n^2 + 1$. That is, for some k we have $(72m + r)k = n^2 + 1$. Then $-1 + (72m+r)k$ is a perfect square for some $k>0$. But this is the same as $-1$ being a quadratic residue $pmod72m+r$.



            My elementary number theory is rusty, but I think that's equivalent to every odd prime divisor of $72m+r$ being of the form $1 pmod4$. So you could try and see whether this applies to all numbers in such a class.







            share|cite|improve this answer














            share|cite|improve this answer



            share|cite|improve this answer








            edited Mar 26 at 13:45

























            answered Mar 26 at 13:01









            Daniel McLauryDaniel McLaury

            16.1k33081




            16.1k33081







            • 1




              $begingroup$
              It is a valiant effort to make sense of the Question, but the considerations you give do not help in determining "density". Showing that certain residues mod $72$ are possible as divisors of numbers of the form $n^2+1$ does not by itself guarantee all such residues as asymptotically equally likely. I applaud your effort to reason on this, but I'm voting to close the Question as the OP has twice asked pretty much the same ill-defined problem.
              $endgroup$
              – hardmath
              Mar 26 at 18:35










            • $begingroup$
              @hardmath lulu in the discussion, understands my question. What is still unclear?
              $endgroup$
              – Arbuja
              Mar 28 at 22:44






            • 1




              $begingroup$
              @Arbuja: With more recent edits to your Question, I'm now able to upvote both it and Daniel's Answer.
              $endgroup$
              – hardmath
              Mar 30 at 0:31












            • 1




              $begingroup$
              It is a valiant effort to make sense of the Question, but the considerations you give do not help in determining "density". Showing that certain residues mod $72$ are possible as divisors of numbers of the form $n^2+1$ does not by itself guarantee all such residues as asymptotically equally likely. I applaud your effort to reason on this, but I'm voting to close the Question as the OP has twice asked pretty much the same ill-defined problem.
              $endgroup$
              – hardmath
              Mar 26 at 18:35










            • $begingroup$
              @hardmath lulu in the discussion, understands my question. What is still unclear?
              $endgroup$
              – Arbuja
              Mar 28 at 22:44






            • 1




              $begingroup$
              @Arbuja: With more recent edits to your Question, I'm now able to upvote both it and Daniel's Answer.
              $endgroup$
              – hardmath
              Mar 30 at 0:31







            1




            1




            $begingroup$
            It is a valiant effort to make sense of the Question, but the considerations you give do not help in determining "density". Showing that certain residues mod $72$ are possible as divisors of numbers of the form $n^2+1$ does not by itself guarantee all such residues as asymptotically equally likely. I applaud your effort to reason on this, but I'm voting to close the Question as the OP has twice asked pretty much the same ill-defined problem.
            $endgroup$
            – hardmath
            Mar 26 at 18:35




            $begingroup$
            It is a valiant effort to make sense of the Question, but the considerations you give do not help in determining "density". Showing that certain residues mod $72$ are possible as divisors of numbers of the form $n^2+1$ does not by itself guarantee all such residues as asymptotically equally likely. I applaud your effort to reason on this, but I'm voting to close the Question as the OP has twice asked pretty much the same ill-defined problem.
            $endgroup$
            – hardmath
            Mar 26 at 18:35












            $begingroup$
            @hardmath lulu in the discussion, understands my question. What is still unclear?
            $endgroup$
            – Arbuja
            Mar 28 at 22:44




            $begingroup$
            @hardmath lulu in the discussion, understands my question. What is still unclear?
            $endgroup$
            – Arbuja
            Mar 28 at 22:44




            1




            1




            $begingroup$
            @Arbuja: With more recent edits to your Question, I'm now able to upvote both it and Daniel's Answer.
            $endgroup$
            – hardmath
            Mar 30 at 0:31




            $begingroup$
            @Arbuja: With more recent edits to your Question, I'm now able to upvote both it and Daniel's Answer.
            $endgroup$
            – hardmath
            Mar 30 at 0:31

















            draft saved

            draft discarded
















































            Thanks for contributing an answer to Mathematics Stack Exchange!


            • Please be sure to answer the question. Provide details and share your research!

            But avoid


            • Asking for help, clarification, or responding to other answers.

            • Making statements based on opinion; back them up with references or personal experience.

            Use MathJax to format equations. MathJax reference.


            To learn more, see our tips on writing great answers.




            draft saved


            draft discarded














            StackExchange.ready(
            function ()
            StackExchange.openid.initPostLogin('.new-post-login', 'https%3a%2f%2fmath.stackexchange.com%2fquestions%2f3163100%2fdescribe-the-natural-density-of-p-which-divides-natural-numbers-of-the-form-n%23new-answer', 'question_page');

            );

            Post as a guest















            Required, but never shown





















































            Required, but never shown














            Required, but never shown












            Required, but never shown







            Required, but never shown

































            Required, but never shown














            Required, but never shown












            Required, but never shown







            Required, but never shown







            Popular posts from this blog

            Triangular numbers and gcdProving sum of a set is $0 pmod n$ if $n$ is odd, or $fracn2 pmod n$ if $n$ is even?Is greatest common divisor of two numbers really their smallest linear combination?GCD, LCM RelationshipProve a set of nonnegative integers with greatest common divisor 1 and closed under addition has all but finite many nonnegative integers.all pairs of a and b in an equation containing gcdTriangular Numbers Modulo $k$ - Hit All Values?Understanding the Existence and Uniqueness of the GCDGCD and LCM with logical symbolsThe greatest common divisor of two positive integers less than 100 is equal to 3. Their least common multiple is twelve times one of the integers.Suppose that for all integers $x$, $x|a$ and $x|b$ if and only if $x|c$. Then $c = gcd(a,b)$Which is the gcd of 2 numbers which are multiplied and the result is 600000?

            Ingelân Ynhâld Etymology | Geografy | Skiednis | Polityk en bestjoer | Ekonomy | Demografy | Kultuer | Klimaat | Sjoch ek | Keppelings om utens | Boarnen, noaten en referinsjes Navigaasjemenuwww.gov.ukOffisjele webside fan it regear fan it Feriene KeninkrykOffisjele webside fan it Britske FerkearsburoNederlânsktalige ynformaasje fan it Britske FerkearsburoOffisjele webside fan English Heritage, de organisaasje dy't him ynset foar it behâld fan it Ingelske kultuergoedYnwennertallen fan alle Britske stêden út 'e folkstelling fan 2011Notes en References, op dizze sideEngland

            Հադիս Բովանդակություն Անվանում և նշանակություն | Դասակարգում | Աղբյուրներ | Նավարկման ցանկ